Change 84cm into millimetres

Answers

Answer 1

Answer:

840 mm

Step-by-step explanation:

multiply by 10

Answer 2

Answer:

840 millimetres

Step-by-step explanation:

To convert cm to mm, multiply the value in cm by 10

84cm x 10 = 840 mm

Hope this helps! <3


Related Questions

Find the length of CE

Answers

Answer:

C. 37.8 units

Step-by-step explanation:

ED = 17/ cos(38°) = 17 / 0.7880 = 21.6 units

DF = 17× tan (38°) = 17× 0.7813 = 13.3 units

CD = 10/13.3 × 21.6 = 16.2 units

so, the length of CE = 21.6+16.2 = 37.8 units

Help please somebody ASAP

Answers

Answer:

[tex]\frac{-2x+11}{(x-4)(x+1)}[/tex]

Step-by-step explanation:

I don't think we can factor this so we'll have to multiply to make the denominators the same

[tex]\frac{3(x+1)}{(x^2-3x-4)(x+1)}-\frac{2(x^2-3x-4)}{(x+1)(x^2-3x-4)}\\\\\frac{3x+3-(2x^2-6x-8)}{(x^2-3x-4)(x+1)}=\frac{-2x^2+9x+11}{(x^2-3x-4)(x+1)}\\-2x^2+9x+11=(x+1)(-2x+11)\\\\x^2-3x-4=(x+1)(x-4)\\\frac{(x+1)(-2x+11)}{(x+1)(x-4)(x+1)}=\frac{-2x+11}{(x-4)(x+1)}[/tex]

Say you buy halibut at $19 per pound . One portion of seared halibut requires 6 ounces of halibut . How much does the halibut for one portion cost ? Round to the nearest cent .

Answers

Answer:

$7.13

Step-by-step explanation:

Given data

Cost of halibut per pound= $19

Let us convert pound to ounces first

1 pound = 16 ounces

Hence 16 ounces will cost $19

           6 ounces will cost x

cross multiply we have

x= 19*6/16

x=114/16

x=$7.13

Hence 6 ounces will cost $7.13

TZ is a midsegment, which of the following statements CANNOT be true

Answers

Answer:

Option C: QT < TR

Step-by-step explanation:

From the triangle, we can see that UX bisects RS into two equal parts and so it is a perpendicular bisector.

TZ Is a mid segment and it means that T bisects QR into 2 equal parts as well as QS into 2 equal parts.

Thus;

QT = QR

And QZ = SZ

So Option C is not correct because QT = QR

The level of significance is the a. same as the p-value. b. maximum allowable probability of Type I error. c. same as the confidence coefficient. d. maximum allowable probability of Type II error.

Answers

Answer:

The level of significance is the

b. maximum allowable probability of Type I error.

Step-by-step explanation:

The significance level provides the maximum probability of rejecting the null hypothesis when it is true.  It is the same as a type I error (also known as false-positive).  This error occurs when a researcher or investigator rejects a true null hypothesis that is supposed to be accepted.  It is the opposite of a type II error (false-negative), which occurs when the researcher fails to reject a false null hypothesis.

Suppose a large telephone manufacturer has a problem with excessive customer complaints and consequent returns of the phones for repair or replacement. The manufacturer wants to estimate the magnitude of the problem in order to design a quality control program. How many telephones should be sampled and checked in order to estimate the proportion defective to within 9 percentage points with 89% confidence

Answers

Answer:

80 telephones should be sampled

Step-by-step explanation:

In a sample with a number n of people surveyed with a probability of a success of [tex]\pi[/tex], and a confidence level of [tex]1-\alpha[/tex], we have the following confidence interval of proportions.

[tex]\pi \pm z\sqrt{\frac{\pi(1-\pi)}{n}}[/tex]

In which

z is the z-score that has a p-value of [tex]1 - \frac{\alpha}{2}[/tex].

The margin of error is of:

[tex]M = z\sqrt{\frac{\pi(1-\pi)}{n}}[/tex]

89% confidence level

So [tex]\alpha = 0.11[/tex], z is the value of Z that has a p-value of [tex]1 - \frac{0.11}{2} = 0.945[/tex], so [tex]Z = 1.6[/tex].

How many telephones should be sampled and checked in order to estimate the proportion defective to within 9 percentage points with 89% confidence?

n telephones should be sampled, an n is found when M = 0.09. We have no estimate for the proportion, thus we use [tex]\pi = 0.5[/tex]

[tex]M = z\sqrt{\frac{\pi(1-\pi)}{n}}[/tex]

[tex]0.09 = 1.6\sqrt{\frac{0.5*0.5}{n}}[/tex]

[tex]0.09\sqrt{n} = 1.6*0.5[/tex]

[tex]\sqrt{n} = \frac{1.6*0.5}{0.09}[/tex]

[tex](\sqrt{n})^2 = (\frac{1.6*0.5}{0.09})^2[/tex]

[tex]n = 79.01[/tex]

Rounding up(as 79 gives a margin of error slightly above the desired value).

80 telephones should be sampled

I feel like it would be 6/10 but that’s not an answer

Answers

Answer:

I think it would be 3/4

Step-by-step explanation:

A small radio transmitter broadcasts in a 69 mile radius. If you drive along a straight line from a city 93 miles north of the transmitter to a second city 78 miles east of the transmitter, during how much of the drive will you pick up a signal from the transmitter?

Answers

Answer:

See Explanation

Step-by-step explanation:

According to the Question,

We have A small radio transmitter that broadcasts in a 69-mile radius. If you drive along a straight line from a city 93 miles north of the transmitter to a second city 78 miles east of the transmitter.

Thus,

The distance that you get reception is the length of the chord created by the intersection of the circle defining the edge of transmission and the line defining the car trip.

 x2 + y2 = 69²               this is the circle

And,

The Transmitter at the origin

City to the north at (0,93)  & City to the east at (78,0)  

the Slope is M=(-93/78)

Intercept is B= y - mx ⇒  93 - (-93/78)(0) = 93

The equation of the line between the cities is y = (-93/78)x + 93

y = -93x/78 + 93           this is the line

Now, Solve the above two Equations

The intersection is gotten from the picture or solving:

x^2 + [(-93/78)*x + 93]^2 = 69^2

on solving we get, the points approximately are: (67.952,11.98 ) and (23.6277, 64.82)

Now,

From the Pythagorean theorem the total distance of the trip is:  

d1 = √(93^2 + 78^2) ≈ 121.37miles  

And the distance when the signal is picked up is:

d2 =√ [(67.952-23.627)^2 + (64.82 - 11.98)^2] ≈ 68.96 miles

You will pick up a signal from the transmitter in (d2/d1)*100 = 56% of the drive.

The light from a lamp creates a shadow on a wall with a hyperbolic border. Find the equation of the border if the distance between the vertices is inches and the foci are inches from the vertices. Assume the center of the hyperbola is at the origin.

Answers

The equation of the hyperbola is,

(x/12)² - 4y²/(527) = 1

The standard equation of the hyperbola is

(x/a)² - (y/b)² = 1

Here (a, 0) and (-a, 0) are vertices and asymptotes y = ± √(b/a)x

Foci are (c, 0) & (-c, 0)

Then a² + b² = c²

Here we have to give that.,

2a = 24

a = 12

And 2c = 7

c = 7/2

Therefore a = 12 and c = 3.5

Substituting a and c in Pythagorean identity;

b² = 527/4

Then, the equation of the hyperbola is

(x/12)² - 4y²/(527) = 1

For further information regarding hyperbolas, kindly refer

brainly.com/question/28989785

#SPJ4

We have b = 0, which implies that the foci coincide with the vertices, making the hyperbola a degenerate case. In this scenario, the equation of the border would be a vertical line passing through the vertices/foci, given by the equation x = ±a.

To find the equation of the hyperbolic border created by the shadow on the wall, we can start by understanding the properties of a hyperbola. A hyperbola is defined as the set of all points such that the difference of the distances from any point on the hyperbola to two fixed points, called the foci, is constant.

Let's label the vertices of the hyperbola as A and B, and the foci as F1 and F2. The distance between the vertices is given as 2a inches, and the foci are located at a distance c inches from the vertices.

Using the given information, we can find the value of a and c. Since the center of the hyperbola is at the origin, the coordinates of the vertices are (±a, 0), and the coordinates of the foci are (±c, 0).

The distance between the foci is given by the equation:

c = √(a^2 + b^2)

We know that the distance between the foci is given as 2c inches, so:

2c = 2√(a^2 + b^2)

Since c is given as a distance from the vertices, we can substitute c = a - b to simplify the equation:

2(a - b) = 2√(a^2 + b^2)

Squaring both sides to eliminate the square root:

4(a - b)^2 = 4(a^2 + b^2)

Expanding the equation:

4(a^2 - 2ab + b^2) = 4a^2 + 4b^2

Simplifying the equation:

4a^2 - 8ab + 4b^2 = 4a^2 + 4b^2

Canceling out the common terms:

-8ab = 0

Dividing by -8:

ab = 0

This implies that either a = 0 or b = 0. However, since a represents the distance between the vertices and b represents the distance between the foci and vertices, we can rule out a = 0 as it would result in a degenerate hyperbola.

for such more question on hyperbola

https://brainly.com/question/16454195

#SPJ8

Find the value of x.

Answers

Answer:

x = 3

Step-by-step explanation:

A midsegment in a trapezoid is formed when one connects the midpoints of the two legs (non-parallel sides) in a trapezoid. The midsegment theorem states that the length of the midsegment is equal to the average of the two bases (that is the parallel sides).

One can apply the midsegment theorem here by stating the following;

[tex]\frac{(YZ)+(TM)}{2}=PW[/tex]

Substitute,

[tex]\frac{23+11x+2}{2}=29[/tex]

Simplify,

[tex]\frac{25+11x}{2}=29[/tex]

Inverse operations,

[tex]\frac{25+11x}{2}=29[/tex]

[tex]25+11x=58\\\\11x = 33\\\\x = 3[/tex]

The salaries of 235 nurses were recorded and analyzed. The analyst later found that the highest salary was incorrectly recorded as 10 times the actual amount. After the error was corrected, the report showed that the corrected value was still higher than any other salary. Which sample statistic must have changed after the correction was made?

Answers

The sample statistic that must have changed after the correction was made is mean. Because mean is based on all the observation in the data. So changing any value in the data will impact mean.

Changing the highest salary in the data will have no impact on median because median lies at the center of data.

Changing the highest salary in the data will have no impact on mode because mode is the most frequently occurring value in the data.

Changing the highest salary in the data will have no impact on minimum because minimum is the smallest value in the data.

Hence the only statistic which will change is mean.

Answer: A-Mean

Step-by-step explanation:

A.) Mean

B.) Median

C.)  Mode

D.)  Minimum

Exactly how many planes contain points J, K, and N?
a - 0
b - 1
c - 2
d - 3

Answers

A- 0 ..... one plane contains k and n but there isn’t a plane that contains all three points

Is a linear model or a quadratic model a better fit? Quadratic model graph quadratic model linear model

Answers

Linear model is a better fit due to my evidence of doing this answer

Round 0.485 to the nearest hundredth

Answers

Decimals start at tenths, then hundredths, then thousandths, and so on. When we round, we look at the place value that is one smaller than the one we want to round to.

So, let's take a look at the thousandths place in 0.485. The value in the thousandths place is 5. When rounding, if the value is 5 or over we round up and if the value is 4 or lower we round down. Since the value in the thousandths place is 5, we will round the hundredths place up one.

0.485 rounded to the nearest hundredth is 0.49

Hope this helps!

Answer:

0.49

Step-by-step explanation:

[tex]0<x<5=[/tex] Round down

[tex]x\geq5=[/tex] Round up

In this case, it's a round up, so the answer would be...

0.49

Hope this helped! Please mark brainliest!

19. Divide 6/13 by 6/12.
A. 12/13
B. 13/12
c. 1/12
D.9/16

Answers

Answer:

12/13 is the answer

Step-by-step explanation:

I think the answer is A . 12/13

The population of watesville decreases at a rate of 1.6% each year if the population was 62,500 in 2015 what will it be in 2021

Answers

Answer:

Step-by-step explanation:

We need to first find the model for this particular situation, knowing that this is an exponential decay problem. The main equation for exponential growth/decay (as far as population goes for our problem) is

[tex]y=a(b)^x[/tex] where a is the initial population, b is the rate of decrease in the population which can also be written as (1 - r), y is the population after a certain amount of time, x, goes by. We will let year 2015 = 0 so year 2021 can = 6. This keeps our numbers lower and doesn't change the answer!

Our initial population in the year x = 0 is 62500. Our rate of decay is

(1 - .016) so our b value is .984

Filling in to find our model:

[tex]y=62500(.984)^x[/tex]

Now we can use that model and sub in a 6 for x to find the population in the year 2021:

[tex]y=62500(.984)^6[/tex] and

y = 62500(.9077590568) so

y = 56734.9 or, rounded to the nearest person, 56735

Carlos has an aquarium which is 45 cm long, 32 cm wide, and 35 cm high. How much water can the aquarium hold?

Answers

Answer:

volume =l×b×h

45cm×32cm×35cm=48,960cm³

45 x 32 x 35 = 50 400 cm^3 = 50.4 litres

Help and explain !!!!!!

Answers

Answer:

x = -4   or   x = 5

Step-by-step explanation:

To solve the absolute value equation

|X| = k

where X is an expression in x, and k is a non-negative number,

solve the compound equation

X = k or X = -k

Here we have |2 - 4x| = 18

In this problem, the expression, X, is 2 - 4x, and the number, k, is 18.

We set the expression equal to the number, 2 - 4x = 18, and we set the expression equal to the negative of the number, 2 - 4x = -18. Then we solve both equations.

2 - 4x = 18  or  2 - 4x = -18

-4x = 16   or   -4x = -20

x = -4   or   x = 5

Answer:

x = -5 . x= 4

Step-by-step explanation:

because |4| = 4 and |-4| = 4

you can see that TWO inputs can get an output of (lets say) 4

The absolute value function can be seen as a function that ignores negative signs

so to get an OUTPUT of "18" using the absolute value function

there are really two ways of getting there

"2-4x = 18"  AND "2-4x = -18"

if you solve both of those you will find that -5 and 4 will

produce the 18 and -18

Four times a number is 88 less than 6 times the number. Find the number.​

Answers

Answer:

44

Step-by-step explanation:

Let x represent the number.

Create an equation, and solve for x:

4x = 6x - 88

-2x = -88

x = 44

So, the number is 44.

The number is 44.

To find the number.

What is arithmetic?

science that deals with the addition, subtraction, multiplication, and division of numbers and properties and manipulation of numbers. Arithmetic is the basics of the abstract science of numbers and operations on them. The formula for any arithmetic sequence is this: an = a1 + d (n - 1).

Given that:

Let x represent the number.

Create an equation, and solve for x:

4x = 6x - 88

-2x = -88

x = 44

So, the number is 44.

Learn more about arithmetic here:

https://brainly.com/question/24163536

#SPJ2

J. Aitchison collected expenditures data for 20 randomly selected single men and 20 randomly selected single women. He uses the data to conduct a hypothesis test to determine if the mean percent of expenditures that goes toward housing (including fuel and light) is different for men and women. What is the correct alternative hypothesis?
a. Md = 0
b. μα = 0
c. ud > 0
d. Opmen — Вwomen
e. Himen > Mwomen
f. Mmen Mwomen

Answers

Answer:

The alternative hypothesis is [tex]H_1: \mu_M - \mu_W \neq 0[/tex], considering M for men and W for women.

Step-by-step explanation:

He uses the data to conduct a hypothesis test to determine if the mean percent of expenditures that goes toward housing (including fuel and light) is different for men and women.

At the null hypothesis, we test if there is not difference, that is, the difference of the mean is 0, so:

[tex]H_0: \mu_M - \mu_W = 0[/tex]

At the alternative hypothesis, we test if there is a difference, that is, the difference of the means is different of 0, so:

[tex]H_1: \mu_M - \mu_W \neq 0[/tex]

Pls could someone help me with this​

Answers

Answer:

- Bar Gaps should be the same

Y-axis up in units of 5 would help out

Step-by-step explanation:

19. Students at a certain school can enroll in one elective course: painting, theater, choir, or band. This two-way frequency
table gives the number of male and female students enrolled in each class.
Male Female Total
Painting 17 16 33
Theater 15
18
33
Choir 21 25 46
Band 28
25
53
Total 81
84
165
Determine the conditional relative frequency that a student in the sample is enrolled in painting given that the student is
female.
O A. 19.0%
O B. 48.5%
O C. 9.7%
O D. 19.8%

Answers

Answer:

19.0%

Step-by-step explanation:

The probability that a student in the sample data is enrolled in painting Given that the student is female is a conditional probability and can be defined as :

Let,

F = Female ; P = painting

P(Painting Given female) = P(P|F) = (PnF) / F

From the table :

(PnF) = 16

F = 84

Hence,

P(P|F) = 16 / 84 = 0.19047 = 0.19047 * 100%

P(P|F) = 19.0%

By recognizing the series as a Taylor series evaluated at a particular value of x, find the sum of each of the following convergent series

1 + 3 + 9/2! + 27/3! + 81/4! + .....

Answers

Answer:

the answer should be e^3

Step-by-step explanation:

i hope it helps you

A courier service company wishes to estimate the proportion of people in various states that will use its services. Suppose the true proportion is 0.06. If 235 are sampled, what is the probability that the sample proportion will differ from the population proportion by greater than 0.04

Answers

Answer:

0.0098 = 0.98% probability that the sample proportion will differ from the population proportion by greater than 0.04

Step-by-step explanation:

To solve this question, we need to understand the normal probability distribution and the central limit theorem.

Normal Probability Distribution:

Problems of normal distributions can be solved using the z-score formula.

In a set with mean [tex]\mu[/tex] and standard deviation [tex]\sigma[/tex], the z-score of a measure X is given by:

[tex]Z = \frac{X - \mu}{\sigma}[/tex]

The Z-score measures how many standard deviations the measure is from the mean. After finding the Z-score, we look at the z-score table and find the p-value associated with this z-score. This p-value is the probability that the value of the measure is smaller than X, that is, the percentile of X. Subtracting 1 by the p-value, we get the probability that the value of the measure is greater than X.

Central Limit Theorem

The Central Limit Theorem establishes that, for a normally distributed random variable X, with mean [tex]\mu[/tex] and standard deviation [tex]\sigma[/tex], the sampling distribution of the sample means with size n can be approximated to a normal distribution with mean [tex]\mu[/tex] and standard deviation [tex]s = \frac{\sigma}{\sqrt{n}}[/tex].

For a skewed variable, the Central Limit Theorem can also be applied, as long as n is at least 30.

For a proportion p in a sample of size n, the sampling distribution of the sample proportion will be approximately normal with mean [tex]\mu = p[/tex] and standard deviation [tex]s = \sqrt{\frac{p(1-p)}{n}}[/tex]

Suppose the true proportion is 0.06.

This means that [tex]p = 0.06[/tex]

235 are sampled

This means that [tex]n = 235[/tex]

Mean and standard deviation:

[tex]\mu = p = 0.06[/tex]

[tex]s = \sqrt{\frac{p(1-p)}{n}} = \sqrt{\frac{0.06*0.94}{235}} = 0.0155[/tex]

What is the probability that the sample proportion will differ from the population proportion by greater than 0.04?

Proportion below 0.06 - 0.04 = 0.02 or above 0.06 + 0.04 = 0.1. Since the normal distribution is symmetric, these probabilities are equal, which means that we can find one of them and multiply by 2.

Probability the proportion is below 0.02.

p-value of Z when X = 0.02. So

[tex]Z = \frac{X - \mu}{\sigma}[/tex]

By the Central Limit Theorem

[tex]Z = \frac{X - \mu}{s}[/tex]

[tex]Z = \frac{0.02 - 0.06}{0.0155}[/tex]

[tex]Z = -2.58[/tex]

[tex]Z = -2.58[/tex] has a p-value of 0.0049.

2*0.0049 = 0.0098

0.0098 = 0.98% probability that the sample proportion will differ from the population proportion by greater than 0.04

In the diagram, WZ=StartRoot 26 EndRoot.

On a coordinate plane, parallelogram W X Y Z is shown. Point W is at (negative 2, 4), point X is at (2, 4), point Y is at (1, negative 1), and point Z is at (negative 3, negative 1).

What is the perimeter of parallelogram WXYZ?

units
units
units
units

Answers

Answer:

[tex]P = 8 + 2\sqrt{26}[/tex]

Step-by-step explanation:

Given

[tex]W = (-2, 4)[/tex]

[tex]X = (2, 4)[/tex]

[tex]Y = (1, -1)[/tex]

[tex]Z = (-3,-1)[/tex]

Required

The perimeter

First, calculate the distance between each point using:

[tex]d = \sqrt{(x_1 - x_2)^2 + (y_1 -y_2)^2[/tex]

So, we have:

[tex]WX = \sqrt{(-2- 2)^2 + (4-4)^2 } =4[/tex]

[tex]XY = \sqrt{(2- 1)^2 + (4--1)^2 } =\sqrt{26}[/tex]

[tex]YZ = \sqrt{(1- -3)^2 + (-1--1)^2 } =4[/tex]

[tex]ZW = \sqrt{(-3--2)^2 + (-1-4)^2 } =\sqrt{26}[/tex]

So, the perimeter (P) is:

[tex]P = 4 + \sqrt{26} + 4 + \sqrt{26}[/tex]

[tex]P = 8 + 2\sqrt{26}[/tex]

Answer:

its D.

Step-by-step explanation:

took test

Helpi
Identify the domain of the function shown in the graph.

Answers

Answer:

D = all reals (or -7 to 7)

Step-by-step explanation:

If the line continues on for infinity, then the domain is all reals, or negative infinity to positive infinity. If the line ends on the graph that we can see, though, the domain would be [-7 , 7]

Domain is all real numbers

Please kindly help

According to a newspaper article 15% more home remodeling was done in 1985 than in 1984. Professionals performed 75% of all remodeling. If $80.4 billion was spent on residential remodeling in 1985 what was the value of the work done by professionals in 1985?
(1) $ 8.4 billion
(2) $12.06 billion
(3) $20.1 billion
(4) $60 billion
(5) $60.3 billion​

Answers

Answer:

(3) $20.1 billion

Step-by-step explanation:

hope it help

Answer:

(5) $60.3 billion​

Step-by-step explanation:

The diameters of bolts produced in a machine shop are normally distributed with a mean of 5.7 millimeters and a standard deviation of 0.08 millimeters. Find the two diameters that separate the top 3% and the bottom 3%. These diameters could serve as limits used to identify which bolts should be rejected. Round your answer to the nearest hundredth, if necessary.

Answers

Answer:

The diameter that separates the top 3% is of 5.85 millimeters, and the one which separates the bottom 3% is of 5.55 millimeters.

Step-by-step explanation:

Normal Probability Distribution:

Problems of normal distributions can be solved using the z-score formula.

In a set with mean [tex]\mu[/tex] and standard deviation [tex]\sigma[/tex], the z-score of a measure X is given by:

[tex]Z = \frac{X - \mu}{\sigma}[/tex]

The Z-score measures how many standard deviations the measure is from the mean. After finding the Z-score, we look at the z-score table and find the p-value associated with this z-score. This p-value is the probability that the value of the measure is smaller than X, that is, the percentile of X. Subtracting 1 by the p-value, we get the probability that the value of the measure is greater than X.

Mean of 5.7 millimeters and a standard deviation of 0.08 millimeters.

This means that [tex]\mu = 5.7, \sigma = 0.08[/tex]

Top 3%

The 100 - 3 = 97th percentile, which is X when Z has a p-value of 0.97, so X when Z = 1.88.

[tex]Z = \frac{X - \mu}{\sigma}[/tex]

[tex]1.88 = \frac{X - 5.7}{0.08}[/tex]

[tex]X - 5.7 = 1.88*0.08[/tex]

[tex]X = 5.85[/tex]

Bottom 3%

The 3rd percentile, which is X when Z has a p-value of 0.03, so X when Z = -1.88.

[tex]Z = \frac{X - \mu}{\sigma}[/tex]

[tex]-1.88 = \frac{X - 5.7}{0.08}[/tex]

[tex]X - 5.7 = -1.88*0.08[/tex]

[tex]X = 5.55[/tex]

The diameter that separates the top 3% is of 5.85 millimeters, and the one which separates the bottom 3% is of 5.55 millimeters.

Jill has 32 crayons. She loses 4 of the crayons. How many are left?

Answers

Answer:

the answer here is d

the answer is d

Answer:

28

Step-by-step explanation:

Total number of crayons = 32

Number of crayons lost = 4

Therefore, number of crayons she is left with is : 32 - 4 = 28

Working :  

    [tex]32\\04 - \\\overline{28}[/tex]

A cyclist completes a journey of 500 m in 22 seconds, part of the way at 10 m/s and the remainder at 50 m/s. How far does she travel at each speed. solve by forming simultaneous equation

Answers

Answer:

150 m at 10 m/s

350 m at 50 m/s

Step-by-step explanation:

x + y = 500

x/10 + y/50 = 22

~~~~~~~~~~~~~~~~~

x + y = 500

5x  + y  = 1100

~~~~~~~~~~~~~~~~

x + y = 500

-5x  - y  = -1100

-4x = -600

x = 150

y = 350

Other Questions
vention 1 of 10These box plots show daily low temperatures for a sample of days in twodifferent townsTWINAM4141Town 11620MID10152025 M3540Degrees (0)Which statement is the most appropriate comparison of the centers?O A. The median temperature for both towns is 20"B. The mean for town A, 30", is greater than the mean for town 8,25"C. The median temperature for both towns is 30'D. The median for town A, 30', is greater than the median for town B,25PREVIOUS9 M Given the net of the rectangular prism, what is its surface area? Please help me ASAP in my final project I am ready to pay 20$ Help please!!!!! Im using Plato The spatial arrangement of lone pairs and bonded pairs around the centralatom is called Read the excerpt from "Finding Unity in the Alabama Coal Mines. The coal companies, in response, recruited workers from as far as New Yorks Ellis Island, where newly arriving immigrants were desperate for jobs. What is the denotation of the word desperate? in need of excited about overlooked for interested in PLEASE HELP ME Write a 150-word paragraph about five of the zaniest and/or most novel ways youve found online that people earn incomes. Keep in mind that the sites should not be corporate sites. We are looking for unique niches. Have fun. We think you will be surprised when you see what people are willing to pay for online. tri t c my i dng Most people with bulimia _____.suffer extreme weight gainsuffer extreme weight lossmaintain a weight below the normal rangemaintain a weight within the normal range 8. The army will hunt _ the militants The U.S. has plans to build a 30-meter telescope, while the Europeans are thinking about a 39-meter telescope. What technological innovation allows astronomers to be thinking about telescopes with that large an aperture Mannix Corporation stock currently sells for $80 per share. The market requires a return of 10 percent on the firm's stock. If the company maintains a constant 6 percent growth rate in dividends, what was the most recent dividend per share paid on the stock Solve the equation 96=3(8x)^(5/3). If a student walked 2 feet straight to the chalk board in 2 seconds andthen walked 2 feet back to his or her original position at his or her desk atthe same speed, what was the student's displacement at 2 secondscompared to 0 seconds?O 6 feetO O feetO2 feetO 4 feet Vocabulary: dipole, dipole-dipole force, dipole-induced dipole force, electronegativity, intermolecular force, ionic bond, London dispersion force, molecule, nonpolar, nonpolar covalent bond, partial charges, polar, polar covalent bond, valence electron Prior Knowledge Questions (Do these BEFORE using the Gizmo.) 1. A big bully is having a tug-of-war with a small child. There is a ball attached to the middle of the rope. Toward whom will the ball move rules of Linking words If I have 25g of Sodium, how much Sodium Chloride will I theoretically create?O 50g NaCl0 58.3g NaClO 63.7g Naci0 35.4g NaCl A sheet of plastic has an area of 35 square cm. If the sheet is 5 cm thick, what is the volume? In New France (Canada), this was the name for the French fur trappers who lived deep in the forests. A. Courer de Bois B. Creole C. Colony D. Renaissance A charge Q exerts a 1.2 N force on another charge q. If the distance between the charges is doubled, what is the magnitude of the force exerted on Q by q